GYN MIDTERM: CHP. 15-19

Ace your homework & exams now with Quizwiz!

Fluid noted anterior to the uterus would most likely be located within the: a. Pouch of Douglas b. Vesicouterine pouch c. Space of Retzius d. Rectouterine pouch

B. Vesicouterine pouch

Pelvic ligament that extends from the lateral surface of the cervix to the lateral fornix of the vagina and houses the uterine vasculature

Cardinal ligament

What ligament extends from the lateral surface of the cervix to the lateral fornix of the vagina and supports the cervix?

Cardinal ligament

What are the sonography findings of a cystic teratoma?

Complex, partially cystic mass in the ovary that includes one or more academic structures that make Shadow,, tip of the iceberg sign only the interior element of the masses seen while the greater part of the mass is obscured by shadowing,, dermoid plug produces posterior shadowing,, dermoid mesh produced by hair and will appear as numerous linear interfaces within the cystic area of this Mass

What are the potential differential gynecologic diagnosis of elevated white blood cell count?

Endometritis, pelvic inflammatory disease

What fibroids usually lead to abnormal uterine bleeding because of their location in relationship to the endometrium?

Intracavitary

What are the sonography appearance of serous cystadenocarcinoma?

Large, multilocular cystic mass, papillary projections and step tations often noted, ascites

What are the sonography findings of endometrioma?

Most often multiple, predominantly cystic mass with low-level internal echoes, May resemble a hemorrhagic cyst, anechoic or complex mostly cystic mass with posterior enhancement and may have a fluid fluid level

What are the potential differential gynecologic diagnosis of precocious puberty?

Ovarian dysgerminoma, ovarian granulosa cell tumor

Peritoneal spaces located posterior to the broad ligament are referred to as the: a. Rectouterine spaces b. Anterior cul-de-sacs c. Lateral cul-de-sacs d. Adnexa

d. adnexa

all of the following are sonographic findings consistent with adenomyosis except: a. diffuse, enlarged uterus b. myometrial cysts c. hypoechoic areas adjacent to the endometrium d. complex adnexal mass

d. complex adnexal mass

the uterus of attachment of the fallopian tubes to the uterus is the: a. fundus b. corpus c. isthmus d. cornua

d. cornua

which of the following would be caused by a large acoustic interface and subsequent production of false echoes? a. posterior shadowing b. acoustic enhancement c. mirror image d. reverberation

d. reverberation

what congenital malformation of the uterus is common and has a clear association with an increases risk for spontaneous abortion? a. anteflexed b. levoverted uterus c. dextroverted uterus d. septate uterus

d. septate uterus

blood within the tube

hematosalpinx

The area within the brain that is located just beneath the hypothalamus that controls and releases the hormones by the anterior pituitary gland

Hypothalamus

A vaginal and Emily in which the hymen has no opening, therefore resulting in an obstruction of the vagina

Imperforate hymen

The term used for the Isthmus of the uterus during pregnancy

Lower uterine segment

The first 4 weeks after birth

Neonatal

What are the potential differential gynecologic diagnosis of elevated serum lactate dehydrogenase?

Ovarian dysgerminoma

- most common benign ovarian tumor - results from retention of unfertilized ovum differentiates into 3 germ cells - complex, partially cystic - bone, hair, fat, cartilage, teeth

cystic/dermoid teratoma

the most superior and widest portion of the uterus is the: a. corpus b. isthmus c. cervix d. fundus

d. fundus

the location of a fibroid within the myometrium is termed: a. submucosal b. intracavitary c. subserosal d. intramural

d. intramural

What day of the menstrual cycle does ovulation occur?

day 14

Around what day is LH produced by the anterior pituitary gland which stimulates ovulation at which time the graafian follicle which is grown ruptures and expels a small amount of fluid and the ovum into the peritoneal?

14

What are the clinical findings of corpus luteum cysts and corpus luteum of pregnancy?

Asymptomatic, pain associated with Hemorrhage and enlargement of cyst, corpus luteum of pregnancy accompanies a pregnancy

What are the clinical findings of endometrioma?

Asymptomatic, pelvic pain, and fertility, dysmenorrhea, menorrhagia, despair Junia, painful bowel movements

The inner mucosal layer of the uterus

Endometrium

What are the sonography findings of ovarian torsion?

Enlarged ovary, enlarged ovary in the presence of multi follicular development, lack of or diminished flow patterns compared with the non affected ovary

The hormone of the anterior pituitary gland that causes the development of multiple follicles on the ovaries

Follicle-stimulating hormone

What are the ovaries stimulated by?

Follicle-stimulating hormone

The functional inner layer of the endometrium that is altered by the hormones of the menstrual cycle

Functional layer

The hormone released by the hypothalamus that stimulates the pituitary gland to release the hormones that regulate the female menstrual cycle

Gonadotropin releasing hormone

What is the typical ovarian flow?

High resistant during menstrual and proliferative phases and low resistance at mid cycle

The Doppler analysis of malignant ovarian masses often reveal what?

Higher diastolic flow velocities because of the abnormal vessels that are created with malignancy, these vessels often lack smooth muscle within their walls and produce a less resistive waveform pattern

How does the normal ovary appear?

Homogeneous with a medium level too low level echogenicity, multiple follicles may be noted during neonatal and pre-pubertal ages

A hormone produced by the trophoblastic cells of the early placenta, may also be used as a tumor marker in non gravid patients and males

Human chorionic gonadotropin

Hormone produced by the trophoblastic cells of the early placenta, may also be used as a tumor marker in non gravid patients and males

Human chorionic gonadotropin

The hormone produced by the trophoblast cells of the early placenta may also be used as a tumor marker and non gravid patients and males

Human chorionic gonadotropin

What are the potential differential gynecologic diagnosis of palpable abdominal Mass?

Leiomyoma, leiomyosarcoma

Which of the following is said to be a *common cause of DUB*?

Polycystic ovary syndrome (PCOS)

which of the following could be described as an infection of the female genital tract that may involve the ovaries, uterus, and/or the fallopian tubes? a. pseudomyxoma peritonei b. pelvic inflammatory disease c. polycystic ovarian disease d. ovarian torsion

b. pelvic inflammatory disease

the layer of the endometrium that is significantly altered as a result of hormonal stimulation during the menstrual cycle is the: a. myometrium b. endometrial cavity c. functional layer d. basal layer

c. functional layer

a 13 year old girl presents to the sonography department with a history of cyclic pain, abdominal swelling, and amenorrhea. sonographically, you visualize an enlarged uterus and a distended vagina that contains anechoic fluid with debris. what is the most likely diagnosis? a. cervical stenosis b. adenomyosis c. endometriosis d. hematocolpos

d. hematocolpos

The left ovarian vein drains directly into the: a. Right renal vein b. Inferior vena cava c. Aorta d. Left renal vein

d. left renal vein

what artifact would be seen posterior to a tooth within a cystic teratoma?

d. shadowing

the breast cancer drug that inhibits the effects of estrogen in the breast is:

d. tamoxifen

which of the following statements is not true concerning transabdominal pelvic imaging? a. transabdominal imaging of the pelvis provides a global view of the entire pelvis b. transabdominal imaging lacks the detail of endovaginal imaging c. obese patients and patients with a retroverted or retroflexed uterus present a unique challenge to the transabdominal technique d. transabdominal imaging is contraindicated for pediatric patients

d. transabdominal imaging is contraindicated for pediatric patients

Ovulation usually occurs on or about day _____ of a 28-day ovarian cycle.

day 14

secondary amenorrhea is characteristically diagnosed in what patients

postmenarchal women who has had at least 12 months w/out menstruation

The corpus luteum that is maintained during an early pregnancy for the purpose of producing estrogen and progesterone

Corpus luteum of pregnancy

What are the potential differential gynecologic diagnosis of constipation or painful bowel movements?

Endometriosis, leiomyoma, leiomyosarcoma, ovarian mucinous cystadenocarcinoma, ovarian mucinous cystadenoma, ovarian serous cystadenocarcinoma, ovarian serous cystadenoma

When the sonographic *three-line sign is present the functional layer* of the endometrium typically appears:

Hypoechoic

A 24-year old female patient presents to the emergency department with severe right lower quadrant pain, nausea and vomiting. The sonographic examination reveals an enlarged ovary with no detectable doppler signal. What is the most likely diagnosis? A. Ovarian cystadenocarcinoma B. Cystic teratoma C. Ovarian torsion D. Endometriosis

C. Ovarian torsion

Which of the following is an estrogen-producing ovarian tumor? A. Cystic teratoma B. Fibroma C. Thecoma D. Endometrioma

C. Thecoma

What are the sonography appearance of mucinous cystadenocarcinoma?

Large, multilocular cystic mass, papillary projections and septations often noted, I could genetic material within the cystic components of the mass,pseudomyxoma peritonei (complex ascites)

What is the most common benign gynecologic tumors in the leading cause of hysterectomy and gynecologic surgery?

Leiomyoma

The second phase of the ovarian cycle days 15 to 28 is termed what phase?

Luteal

What are the potential differential gynecologic diagnosis of vaginal discharge?

Pelvic inflammatory disease

The uterine body tilts backward and comes in contact with the cervix for me an acute angle between body and cervix

Retroflexion

The invasion endometrial tissue into the myometrium of the uterus is referred to as: a. amenorrhea b. endometriosis c. adenomyomatosis d. adenomyosis

d. adenomyosis

which of the following would typically not be associated with amenorrhea? a. Asherman syndrome b. polycystic ovarian disease c. pregnancy d. adenomyosis

d. adenomyosis

What other term is used to describe the space of Retzius? a. Posterior cul-de-sac b. Anterior cul-de-sac c. Murphy pouch d. Retropubic space

d. retropubic space

- May occur if graafian follicle fails to ovulate - 3cm-8cm - anechoic, thin wall

follicular cyst

Denotes appearance of mass when only ant element is seen

"tip of the iceberg"

The *average menstrual cycle lasts*:

28 days

What are the potential differential gynecologic diagnosis of pelvic pressure and or tenderness?

Adenomyosis, endometritis, leiomyoma, leiomyosarcoma, ovarian mucinous cystadenocarcinoma, ovarian mucinous cystadenoma, ovarian serous cystadenocarcinoma, ovarian serous cystadenoma, pediatric Hydrocolpos or hematocolpos, pelvic inflammatory disease

The area located posterior to the broad ligament, adjacent to the uterus, which contains the ovaries and fallopian tubes

Adnexa

What are the clinical findings of granulosa cell tumors?

Adolescent - pseudoprecocious puberty,, Reproductive ages and postmenopausal women will have abnormal vaginal bleeding

Asymptomatic what are the sonography findings of a nabothian cyst?

Anechoic Mass within the cervix, maybe multiple, maybe complex

What are the clinical findings of a krukenberg tumor?

Asymptomatic, possible weight loss, pelvic pain

The pelvic ligament that provides support to the ovary and extends from the ovary to the lateral surface of the uterus is the: a. Cardinal ligament b. Ovarian ligament c. Broad ligament d. Suspensory ligament of the ovary

B) Ovarian ligament

Which of the following are the paired anterior abdominal muscles that extend from the xiphoid process of the sternum to the pubic bone? a. Iliopsoas muscles b. Rectus abdominis muscles c. Obturator interni muscles d. Piriformis muscles

B) rectus abdominis muscles

The sonographic appearance of an ovarian dermoid tumor in which on the anterior elements of the mass can be seen, while the greater part of the mass is obscured by shadowing is consistent with: A. Whirlpool sign B. Tip of the iceberg sign C. Dermoid mesh sign D. Dermoid plug sign

B. Tip of the iceberg sign

The abdominal aorta bifurcates into the: a. Internal iliac arteries b. Common iliac arteries c. Ovarian arteries d. External iliac arteries

B. common iliac arteries.

The corpus luteum that is maintained during early pregnancy for the purpose of producing estrogen and progesterone

Corpus luteum of pregnancy

Which of the following sonographic findings would not increase the likelihood of an ovarian malignancy? A. Septation measuring >3mm in thickness B. Irregular borders C. Solid wall nodule D. Anechoic components with acoustic enhancement

D. Anechoic components with acoustic enhancement

A change in menstrual bleeding patterns related to endocrine abnormalities where hormonal imbalances result in endometrial changes in subsequent abnormal bleeding

Dysfunctional uterine bleeding

*Painful and difficult menstruation* is termed:

Dysmenorrhea

Difficult or painful menstruation

Dysmenorrhea

Painful and difficult menstruation

Dysmenorrhea

The outer germ cell layer of the embryo that develops into the skin, hair, and nails, and other structures

Ectoderm

What structure may be noted on the ovary just *prior to ovulation*?

Graafian follicle

What are the muscles visualized on a sonogram?

Rectus abdominis,iliopsoas, obturator internus, piriformis, levator ani, coccygeus

An ultrasound procedure that uses saline instillation into the endometrial cavity and Fallopian tubes to evaluate for internal abnormalities

Sonohysterography

Extraperitoneal space located between the bladder and symphysis pubis that contains fat

Space of retzius

The arteries within the *functional layer* of the endometrium that are altered by the hormones of the ovary and are shed with menstruation are the:

Spiral arteries

Denotes the sonography tech appearance of a cystic teratoma when only the anterior element of the mass is seen, while the greater part of the mass is obscured by shadowing

Tip of the iceberg sign

Describe the condition of stage 3 of ovarian carcinoma

Tumor involves one or both ovaries with confirmed peritoneal Mets outside of the pelvis and or Regional lymph node involvement

Describe the condition of stage 1 ovarian carcinoma?

Tumor is confined to the ovary

What are the clinical findings of adenomyosis?

Uterine enlargement, dysmenorrhea, menometrorrhagia, pelvic pain, despareunia, multiparous

Following the neonatal period the cervical anterioposterior is equal to or slightly greater than what?

Uterine fundus

A patient presents to the sonography department with a history of uterine prolapsed. Which of the following best describes this disorder? a. A condition that results from the weakening of the pelvic diaphragm muscles and allows for the displacement of the uterus, often through the vagina. b. A congenital anomaly that results in the duplication of the uterus. c. A condition that results in the abnormal invasion of the myometrium through the bladder wall leading to hematuria. d. An abnormality that describes the inversion of the myometrium and endometrium.

a condition that results from the weakening of the pelvic diaphragm muscles and allows for the displacement of the uterus, often through the vagina

which of the following statements would be considered an acceptable disadvantage of endovaginal imaging? a. endovaginal imaging has a limited field of view b. the resolution of endovaginal imaging is reduced compared to transabdominal imaging c. endovaginal imaging is more time consuming than transabdominal imaging d. endovaginal imaging can be performed only by female sonographers

a. endovaginal imaging has a limited field of view

The uterine artery branches off of the: a. Abdominal aorta b. Uterine plexus c. Internal iliac artery d. External iliac artery

c. internal iliac artery

sonongraphic appearance of endometrioma

cystic mass with low level echoes resembling a hemorrhagic cyst

what laboratory value would be most useful to evaluate in a patient with suspected internal hemorrhage?

d. hematocrit

Pelvic bones, when visualized on sonography, will appear: a. Anechoic b. Hypoechoic c. Dark d. Hyperechoic

d. hyperechoic

sonographic appearance of torsed ovary

enlarged ovary, with or without presence of multifollicular development

initiates proliferation and thickening of endo

estrogen

what are the 2 ovarian phases

follicular and luteal

malignant ovarian tumor metastasized from GI tract, smooth walled hypoechoic tumor, commonly bilateral and may be accompanied by ascites

krukenberg tumor

After the graafian follicle ruptures, the remaining structure is termed the:

corpus luteum

Progesterone is produced by

corpus luteum

malignant, less often bilateral, associated with pseudomyxoma

cystadenocarcinoma

The paired embryonic ducts that develop into the female urogenital tract are the: a. fallopian ducts b. wolffian ducts c. gartner ducts d. mullerian ducts

d. mullerian ducts

The muscles that may be confused with the ovaries on a pelvic sonogram include the: a. Rectus abdominis and obturator internus muscles b. Levator ani and coccygeus muscles c. Obturator internus and levator ani muscles d. Piriformis and iliopsoas muscles

d. piriformis and iliopsoas muscles

LH surges on what day? and results in what?

day 14; ovulation

average endo thickness on day 4 and 8

day 4 = 4mm day 8 = 8mm

contains various tissues causing shadowing

dermoid plug

double layer thickness includes what

distance from basal layer to basal layer

patients with granulosa cell tumors have a 10-15% chance of devloping what due to consistent estrogen stimulation

endometrial carcinoma

- benign hemorrhagic tumor that forms from implantation of ectopic endo tissue - functional endo tissue causes hormones to act as it is uterus causing hemorrhage

endometrioma

ovarian tumor that has a high incidence of being malignant. most common cancer to originate within endometrioma. Appears as a complex mass w/ solid components

endometroid tumor

the cells surrounding the follicle produces what

estrogen

as endocrine glands, the ovaries are responsible for producing what hormones

estrogen and progesterone

the ovary produces what two hormones

estrogen and progesterone

what endo layer changes throughout menstruation, location of placenta

functional layer

hormone produced by the hypothalamus

gonadotropin-releasing hormone

- sex cord stromal tumor - most common estrogenic tumor - unilateral - postmenopausal women

granulosa cell tumors

Which hormone *maintains the corpus luteum* during pregnancy?

hCG

follicular cyst that contains blood, appears complex or completely echogenic

hemorrhagic cyst

doppler analysis of malignant ovarian masses often reveal what

high diastolic flow velocities because of abnormal vessels created with less resistance.

typical ovarian flow during menstrual and proliferative phase

high resistance - menstrual low resistance - midcycle

The normal sonographic appearance of the ovary

homogenous with medium to low level echogenicity

blastocyst produce what hormone

human chorionic gonadotropin (hCG)

patients with theca lutein cysts may suffer from

hyperemesis and pelvic fullness

may cause follicular cyst

hyperstimulation of ovary

distal portion of tube that provides an opening to peritoneal cavity

infundibulum

within the cornu of the uterus lies the _________ extension of the fallopian tube

intramural

means "bridge". Short and narrow segment of the tube connecting interstitial area of ampulla

isthmus

pituitary gland is also known as what

master gland

means middle pain, pain at time of ovulation

mittleschmerz

often larger , tend to have septations but not usually bilateral, INTERNAL DEBRIS

mucinous cystadenoma

signs and symptoms of ovarian torsion

nausea, vomiting, abdominal pain

ovarian fossa includes what

outer cortex and inner medulla

three layers of the fallopian tube

outer serosa, middle muscular layer, inner mucosa

- Mass of ovary - Site of oogenesis

ovarian cortex

located posterior to ureter and internal iliac artery. Superior to external iliac artery

ovarian fossa

if corpus luteum cyst become too large it can lead to

ovarian torsion

reoccurs most often on right ovary, most common cause being ovarian cyst or mass

ovarian torsion

paired, oval shaped intraperitoneal organs with a dual blood supply

ovaries

small cysts adjacent to ovary, may have septations, larger may cause torsion

paraovarian cyst

follicular phase of ovarian cycle

period of follicle growth (days 1-14)

patients with dysgerminoma may present with what

precocious puberty and elevate hCG.

infection within the tube often caused by PID

salpingitis

sex cord stromal ovarian neoplasm associated with virilization. May present with hirsutism. Often women <30yrs, may be malignant, may appear as solid hypoechoic masses or complex cystic mass

sertoli - leydig cell tumor

The trophoblastic cells surrounding the Blastocyst that are responsible for producing human chorionic gonadotropin

syncytiotrophoblastic cells

When a Graafian follicle ruptures it forms what

the corpus luteum

the one follicle remaining on the ovary turns into what

the graafian follicle/dominant follicle

All Venus structures mirror their arterial counterparts with the exception of what?

the left ovarian vein drains into the left renal vein

- largest and least common cyst - elevated hCG levels (>100,000) - common with gestation trophoblastic - multiple gestation increases risk

theca lutein cyst

- benign ovarian sex cord stromal tumor - most common in postmenopausal women - estrogen producing tumors - hypoechoic, solid mass w/post attenuation - unilateral

thecoma

The urinary bladder, uterus, and ovaries are located within the: a. True pelvis b. False pelvis

true pelvis

A benign smooth muscle tumor of the uterus AKA fibroid AKA uterine Myoma

Leiomyoma

The malignant manifestation of a leiomyoma

Leiomyosarcoma

The *second phase of the ovarian cycle* is called the:

Luteal phase

Developing into cancer

Malignant degeneration

Ascites and pleural effusion in the presence of some benign ovarian tumors

Meigs syndrome

The *first menstrual cycle* is termed:

Menarche

Abnormally heavy and prolonged menstration

Menorrhagia

Abnormally heavy and prolonged menstruation

Menorrhagia

The germ cell layer of the embryo that develops into the circulatory system, muscles, reproductive system, and other structures

Mesoderm

Irregular menstrual bleeding between periods

Metrorrhagia

Which is larger serous or mucinous cystadenoma?

Mucinous

Which malignant ovarian tumor is often associated with pseudomyxoma peritonei?

Mucinous cystadenocarcinoma

Paired embryonic ducks that develop into the female urogenital tract

Mullerian ducts

Having more than one internal cavity

Multiloculated

Having birth more than one child

Multiparous

The muscular layer of the uterus

Myometrium

Who are at a greater risk for development of fibroids?

Obese, black, non-smokers, perimenopause

The creation of an ovum

Oogenesis

A syndrome resulting from hyperstimulation of the ovaries by fertility drugs, results in the development of multiple, enlarged follicular ovarian cysts

Ovarian hyperstimulation syndrome

What are the potential differential gynecologic diagnosis of oliguria?

Ovarian hyperstimulation syndrome

What are the potential differential gynecologic diagnosis of ovarian enlargement?

Ovarian hyperstimulation syndrome

What are the potential differential gynecologic diagnosis of nausea and vomiting?

Ovarian hyperstimulation syndrome, ovarian torsion

Massive amounts of pelvic ascites may be associated with what?

Ovarian tumors, cirrhosis, portal hypertension

Pseudoprecocious puberty has been linked with what?

Ovarian, adrenal, and liver tumors

Following ovulation the ovary Begins the what phase while the endometrium enters the what phase?

Ovary Begins the luteal phase while the endometrium enters the secretory phase

Fallopian tubes AKA

Oviducts, uterine tubes, salpiges

The release of the mature egg from the ovary

Ovulation

When is Meg syndrome present?

Pelvic ascites is accompanied by pleural effusion and a benign ovarian Mass

Salpingitis is often caused by what?

Pelvic inflammatory disease

What are the clinical findings of leiomyosarcoma

Pelvic pressure, menorrhagia, palpable abdominal Mass, enlarged bulky uterus, urinary frequency, dysuria, constipation, infertility

What are the clinical findings of the uterine leiomyoma?

Pelvic pressure, menorrhagia, palpable pelvic Mass, enlarged bulky uterus, urinary frequency, dysuria, constipation, infertility

Mullerian ducts AKA?

Perimesonephric ducts

What are the potential differential gynecologic diagnosis of hirsutism?

Polycystic ovarian disease, sertoli leydig cell tumor

If a patient has menstrual cycles that are 17 days apart, what would you describe this as?

Polymenorrhea

What is defined as frequent *regular cycles but less than 21 days apart*?

Polymenorrhea

The vagina is positioned where to the urethra?

Posterior

Where is the ovarian fossa located?

Posterior to the ureter and internal iliac artery and superior to the external iliac artery

Endometrial atrophy is a common occurrence in What patients?

Postmenopausal who presented with vaginal bleeding

Failure to experiencing menarche before age 16

Primary amenorrhea

A hormone that prepares the uterus for the pregnancy, maintains pregnancy, and promotes development of the mammary glands primarily produced by the ovaries and placenta

Progesterone

The *corpus luteum primarily releases*:

Progesterone

What maintains the thickness of the endometrium in preparation for implantation?

Progesterone

A condition that results from the weakening of the pelvic diaphragm muscles and allows for displacement of the uterus, often through the vagina

Prolapse

The multiplication of similar forms

Proliferation

The endometrial cycle consists of what two phases?

Proliferative and secretory

The presence of pus within the fallopian tube

Pyosalpinx

A diagnostic imaging modality that uses ionizing radiation for Imaging bones, organs, and some soft tissue structures

Radiography

What type of uterus makes a challenge for transabdominal technique?

Retroverted or retroflexed

Caused by large acoustic interface and subsequent production of false echoes, echogenic region in the anterior aspect of the urinary bladder

Reverberation

Ovarian torsion often occurs on which side? What is the most common cause?

Right, ovarian cyst or mass such as the benign cystic teratoma or paraovarian cyst

What ligaments extends from the uterine cornua to labia majora between the folds of the broad ligament and support the uterine fundus?

Round ligament

Characteristically diagnosed in the postmenarcheal woman who has had at least 12 months without a menstrual cycle

Secondary amenorrhea

What phase of the endometrial cycle occurs after ovulation and is stimulated by progesterone?

Secretory

When the ovary is in the *luteal phase*, the endometrium is in the:

Secretory

The *second phase* of the endometrial cycle is the:

Secretory phase

The second phase of the endometrial cycle

Secretory phase

Congenital malformation of the uterus that results in a single septum that separates two endometrial cavities

Septate uterus

A partition separating two or more cavities

Septations

During the late proliferative phase or Periovulatory phase endometrial layer displays what?

Stark contrast and can measure between 6 and 10 mm the outer academic basal layer surrounds the more hypoechoic functional layer while the functional layer is separated by the echogenic endometrial stripe AKA three lines sign

What is the primary role of gonadotropin-releasing hormone?

Stimulates release of hormones by the anterior pituitary gland.

Krukenberg tumor is a malignant ovarian tumor that metastasized from the GI tract most frequently the what?

Stomach

A leiomyoma that distorts the shape of the endometrium

Submucosal

Congenital malformation of the uterus that results in abnormal uterine Contour with an endometrium that branches into two horns

Subseptate uterus

What ligament extends from the ovaries to the pelvic sidewalls and supports ovaries and tubes?

Suspensory ligament of the ovaries, infundibulopelvic

Pelvic ligament that provides support to the ovary and extends from the ovaries to the pelvic sidewalls

Suspensory ligament of the ovary

A breast cancer drug that inhibits the effects of estrogen in the breast

Tamoxifen

What happens to the corpus luteum if there is no implantation?

The corpus luteum becomes the corpus albicans due to a decrease in estrogen

Which of the following is a true statement with respect to fallopian tubes?

The fallopian tube is usually only identified if there is an obstruction and it is distended with fluid.

What signals the corpus luteum to continue to produce progesterone when a fertilized egg is approaching the endometrium?

The syncytiotrophoblast cells produce hCG which keeps the corpus luteum producing progesterone.

Cells of the follicle that produce estrogen

Theca internal cells

A 60-year-old female patient presents with elevated estrogen levels, ovarian tumor, ascites, and pleural effusion. What is the most likely diagnosis for the ovarian tumor?

Thecoma

Which of the following is false with respect to theca lutein cysts?

They are found when hCG levels are decreased.

Congenital malformation of the uterus that results in the uterus with one horn

Unicornuate uterus

What are the sonography findings of a Brenner tumor?

Unilateral, small, solid, hypoechoic Mass, may contain calcifications

Having only one internal cavity

Unilocular

Branches of the internal iliac artery that supplies blood to the uterus, ovaries, and fallopian tubes

Uterine arteries

Besides a hysterectomy or myomectomy what is an alternative treatment for fibroids?

Uterine artery embolization

Procedure used to block the blood supply to a leiomyoma

Uterine artery embolization

What ligament extends from the uterus to the sacrum and supports the uterus?

Uterosacral ligament

a 38 year old female patient presents to the sonography department for a pelvic sonogram with an indication of pelvic pain. upon sonography interrogation, the sonographer notes an anechoic mass within the cervix. this is most likely represents a: a. nabothian cyst b. benign follicular cyst c. dermoid cyst d. gartner duct cyst

a. nabothian cyst

which of the following fibroid locations would most likely result in abnormal uterine bleeding because of its relationship to the endometrium? a. submucosal b. intramural c. subserosal d. subserosal pedunculated

a. submucosal

may be caused by number of complications and defined as a change in menstrual bleeding

abnormal uterine bleeding

rare cancer of the fallopian tubes

adenocarcinoma

The longest and most tortuous segment of the Fallopian tube is the. Location of fertilization

ampulla

endometrioma can be located where

anywhere outside of the endometrial cavity

The right ovarian artery branches off of the: a. Aorta b. Right renal artery c. Uterine artery d. Internal iliac artery

aorta

a 24 year old female patient presents to the sonography department for a pelvic sonogram with an indication of pelvic pain. upon sonographic interrogation, the sonographer notes an anechoic mass within the vagina. this mass most likely represents a. nabothian cyst b. gartner duct cyst c. dandy walker cyst d. ovarian cyst

b. gartner duct cyst

A simple fluid accumulation within the vagina secondary to an imperforate hymen is: a. hydrometrocolpos b. hydrocolpos c. hematometra d. hematocolpos

b. hydrocolpos

if fertilization does not occur what happens to the corpus luteum

becomes corpus albicans

the recesses of the vagina are the: a. cornu b. isthmi c. fornices d. parity

c. fornices

another name for endometrioma

chocolate cyst

primary role of the fimbria

draw unfertilized egg into tube

most common malignant germ cell tumor of ovary. Patients <30

dysgerminoma

contains ovarian vasculature and lymphatics

medulla

first menstrual cycle is termed

menarche

days 1 through 5 of menstrual cycle correlate with what

menses

corpus luteum produces what during the luteal phase

progesterone

maintains thickness of endo for implantation

progesterone

what are the 2 uterine phases

proliferative and secretory

the fallopian tube may become distended when

secondary to obstruction or infection

two types of cystadenoma

serous and mucinous

all of the following are common indications for a pelvic sonogram except: a. evaluation of congenital anomalies b. evaluation of pelvic anatomy immediately following a motor vehicle accident c. localization of an intrauterine contraceptive device d. postmenopausal bleeding

!!! b. evaluation of pelvic anatomy immediately following a motor vehicle accident

all of the following would be relevant laboratory tests to evaluate before performing a routine pelvic sonogram except: a. human chorionic gonadotropin b. hematocrit c. white blood cell count d. lipase

!!! d. lipase

The area located posterior to the broad ligament, adjacent to the uterus, which contains ovaries and fallopian tubes

Adnexa

What are the sonography findings of hydrosalpinx?

Anechoic

What are double folds of the peritoneum?

Broad ligament, suspensory ligament of the ovary

Areas just inferior to the fundus of the uterus where the Fallopian tubes attached bilaterally

Cornua

The uterine body

Corpus

The basal layer is slightly altered during the menstrual cycle and consist of what?

Dense cellular stroma

may be idiopathic, related to hormonal imbalances resulting in endo changes. Ex: PCOS

Dysfunctional uterine bleeding

The germ cell layer of the embryo that develops into the GI and respiratory tracts

Endoderm

What are the potential differential gynecologic diagnosis of postmenopausal vaginal bleeding?

Endometrial a trophy, endometrial hyperplasia, ovarian granulosa cell tumor, ovarian mucinous cystadenocarcinoma, ovarian mucinous cystadenoma, ovarian serous cystadenocarcinoma, ovarian serous cystadenoma, ovarian thecoma

Fluid accumulation within the vagina

Hydrocolpos

The abnormal accumulation of fluid within the fallopian tube

Hydrosalpinx

A radiographic procedure that uses a Dye instilled into the endometrial cavity and the Fallopian tubes to evaluate for internal abnormalities

Hysterosalpingography

Location of leiomyoma within the myometrium of the uterus

Intramural

An enzyme found within the blood that may be used to monitor renal function, may also be used as a tumor marker for some ovarian tumors

Lactate dehydrogenase

Destruction or breakdown

Lysis

Pain at the time of ovulation

Middleschmertz

Produced by a strong reflector and results in a copy of the anatomy being placed deeper than the correct location

Mirror image

A patient presents with mittelschmerz. Sonography demonstrated a small amount of free fluid in the right adnexa. What is the likely cause

Ovulation

A small protrusion of tissue

Papillary projections

leiomyoma that extends from the uterus on a stock

Pedunculated uterine leiomyoma

Group of pelvic muscles consisting of the levator Ani and coccygeus muscles that provide support to the pelvic organs

Pelvic diaphragm

Granulosa cell tumors are most commonly found in whom?

Postmenopausal but could be found in younger patients

Defined as pubertal development for the age of eight, the early development of pubic hair, breasts, or genitals

Precocious puberty

the pubertal development before the age of 8, the early development of pubic hair, breasts, or genitals

Precocious puberty

What *hormone maintains the thickness* of the endometrium after ovulation?

Progesterone

Which of the following *hormones is released by the ovary* during the *second half* of the menstrual cycle?

Progesterone

What phase of the endometrial cycle occurs after menstruation and lasts until ovulation?

Proliferative

The first phase of the *endometrial cycle* is the:

Proliferative phase

The first phase of the endometrial cycle

Proliferative phase

What is the primary function of the uterus?

Provide a place for the products of conception to implant and develop

What is the primary purpose of the fallopian tube?

Provide an area for fertilization and to offer a means of transportation for the products of conception to reach the uterine cavity

Secondary sexual development induced by sex steroids or from other sources like ovarian tumors, adrenal tumors or steroid use

Pseudoprecocious puberty

Secondary sexual development induced by sex steroids or from other sources like ovarian tumors, adrenal tumors, or steroid use

Pseudoprecocious puberty

Arteries that Supply blood to the deeper layers of the myometrium

Radial arteries

How is a yolk Sac tumor characterized?

Rapid growth

What are the sonography findings of leiomyosarcoma?

Rapidly growing Mass within the uterus, hypoechoic Mass within the uterus, posterior shadowing from the mass, degenerating fibroids may have calcifications or cystic components, multiple fibroids appear has an enlarged irregular shaped diffusely heterogeneous uterus

Peritoneal outpouching located between the uterus and rectum AKA posterior cul-de-sac and pouch of Douglas

Rectouterine pouch

What pelvic muscles are anterior?

Rectus abdominis

What is the responsibility of the hypothalamus?

Regulate and release of hormones by the anterior pituitary gland the hypothalamus achieves this function by releasing its own hormone gonadotropin-releasing hormone which in turn stimulates the release of hormones by the anterior pituitary gland

What are the ovaries responsible for?

Releasing estrogen and progesterone throughout the menstrual cycle

Endometrioma are found in whom?

Reproductive years

A sonogram of a postpubertal woman demonstrated a 5-cm simple cyst on the left ovary. A follow-up exam 3 months later demonstrates a normal ovary. What do these findings likely represent?

Resolution of a follicular retention cyst

What do dermoids result from?

Retention of an unfertilized ovum that differentiates into the three germ cell layers

The uterine body tilts backward without a Bend where the cervix and body meet

Retroversion

The posterior border of the pelvic cavity is marked by what?

Sacrum and coccyx

What bones Mark the boundaries of the pelvic cavity?

Sacrum, coccyx, and innominate bones

The Bony pelvis consists of what?

Sacrum, coccyx, innominate bones

Tumor marker that is elevated in the presence of an ovarian dysgerminoma and other abdominal abnormalities

Serum lactate dehydrogenase

Caused by attenuation of the sound beam, seen posterior to the dense structures like pelvic bones or teeth within a dermoid

Shadowing

These 1 through 5 of the menstrual cycle correlate with Menses at which this time the endometrium is what?

Shed

What are the sonography findings of a follicular cyst?

Simple cyst - anechoic, thin walled, unilocular, round, posterior enhancement Hemorrhagic cyst - variable appearances including complex components or entirely echogenic depending on the amount of blood in the stage of lysis

What is the sonography appearance of Fallopian tube carcinoma?

Solid Mass within the adnexa, tube may become distended secondary to obstruction or infection, fluid contained within the distended tubes could be simple serous fluid, blood, or pus

Occlusion or in perforation of the vagina, can be congenital or acquired

Vaginal atresia

With congenital malformation of the vagina the obstructions can be the result of what?

Vaginal atresia, vaginal septum, imperforate hymen

Recesses of the vagina

Vaginal fornices

Peritoneal outpouching located between the bladder in the uterus AKA anterior cul-de- sac

Vesicouterine pouch

Female changes within the female that are caused by increased androgens, may lead to deepening of the voice and hirsutism

Virilization

What are the clinical findings of sertoli leydig cell tumors?

Virilization, abnormal menstruation, hirsutism

Do dermoids have the capability of malignant degeneration?

Yes

Ovary malignant germ cell tumor of the ovary

Yolk Sac tumor

Sertoli leydig tumors are found more often in what women?

Younger than 30

Dysgerminoma arise more often in whom?

Younger than 30 and may be found in pregnancy

a patient presents to the sonography department for a pelvic sonogram with a history of adenomyosis that was diagnosed following an MRI of the pelvis. what are the most likely sonographic findings? a. complex, bilateral adnexal masses b. myometrial cysts with enlargement of the posterior uterine wall c. endometrial thinning and cervical dilation d. uterine atrophy with bilateral ovarian cysts

b. myometrial cysts with enlargement of the posterior uterine wall

Both the straight and spiral arteries are branches of the: a. Common iliac artery b. Radial artery c. Arcuate artery d. External iliac artery

b. radial artery

which of the following would be most indicative of a leiomyosarcoma? a. vaginal bleeding b. rapid growth c. dysuria d. large hypoechoic mass

b. rapid growth

The uterine arteries supply blood to all of the following except: a. Fallopian tubes b. Rectum c. Ovaries d. Uterus

b. rectum

The sonographic pelvic examination of a female patient reveals an extensive amount of ascites. In the transverse plane, you visualize two echogenic structures extending from the side walls of uterus to the pelvic side walls bilaterally. These structures are most likely the: a. Broad ligaments b. Cardinal ligaments c. Ovarian ligaments d. Uterosacral ligaments

broad ligaments

all of the following are proper techniques for providing patient care for patients during a pelvic sonogram except: a. all transducers and their cords should be cleaned before performing a pelvic sonogram b. endovaginal transducers should be cleaned with a high-level disinfectant c. a probe cover should be placed on the transducer for transabdominal imaging to prevent the spread of infection d. sterile jelly should be used as a lubricant for endovaginal imaging

c. a probe cover should be placed on the transducer for transabdominal imaging to prevent the spread of infection

abscense of menstruation is referred to as: a. dysuria b. dysmenorrhea c. amenorrhea d. menorrhagia

c. amenorrhea

the normal position of the uterus is: a. retroverted b. retroflexed c. anteverted d. dysverted

c. anteverted

the rigid region of the uterus located between the vagina and the isthmus is the: a. cornu b. corpus c. cervix d. fundus

c. cervix

difficult or painful intercourse is referred to as: a. dysuria b. dysmenorrhea c. dyspareunia d. hydrpcolpos

c. dyspareunia

which of the following is defined as pain during intercourse?

c. dyspareunia

The vagina is located __ to the uterus: a. anterior b. posterior c. inferior d. medial

c. inferior

what abnormality results from the ovary twisting on its mesenteric connection?

c. ovarian torsion

Another name for the rectouterine pouch is the: a. Space of Retzius b. Pouch of Retzius c. Pouch of Douglas d. Anterior cul-de-sac

c. pouch of douglas

What is considered the most dependent part of the peritoneal cavity? a. Space of Retzius b. Anterior cul-de-sac c. Pouch of Douglas d. Rectovessicular pouch

c. pouch of douglas

the "s" in the STAR criteria stand for:

c. smooth walls

The ligament that houses the vasculature of the uterus is the: a. Cardinal ligament b. Ovarian ligament c. Broad ligament d. Suspensory ligament of the ovary

cardinal ligament

the outer layer of the endometrium is the: a. myometrium b. endometrial cavity c. functional layer d. basal layer

d. basal layer

hemorrhage from endometriosis forms into focal areas of bloody tumors called

endometrioma

simple serous fluid within the tube

hydrosalpinx

area within the brain beneath the thalamus

hypothalamus

second phase of the ovarian cycle

luteal phase, days 15 - 28

caused by congenital abnormalities or congenital obstructions

primary amenorrhea

pus in tube

pyosalpinx

dermoids are commonly found in what patients

reproductive age and postmenopausal

cells surrounding blastocyst

syncytotrophoblast

second most common malignant germ cell tumor. Characterized by rapid growth. Female <20. Highly malignant. Elevated AFP.

yolk sac tumor

Fertilization typically occurs on what day?

15

the average menstrual cycle lasts how long

28 days

Approximately what percent of serous cystadenoma are benign occurring more often in women in their 40s and 50s as well as during pregnancy?

50 to 70

How long are the Fallopian tubes?

7 to 12 centimeter

Which of the following best describes the normal appearance of a normal ovary?

<15 mL volume, hypoechoic to myometrium with multiple anechoic follicles

The cystic mass commonly noted with pregnancy is the: A. Corpus luteum B. Dermoid cyst C. Dysgerminoma D. Serous cystadenoma

A. Corpus luteum

The dominant follicle prior to ovulation is termed the: A. Graafian follicle B. Corpus albicans C. Corpus luteum D. Medulla

A. Graafian follicle

Paraovarian cysts are small cysts located where?

Adjacent to ovary

The uterine body tilts forward and comes in contact with a cervix forming an acute angle between body and cervix

Anteflexion

What are the clinical findings of paraovarian cyst?

Asymptomatic, if cyst is enlarged patient may present with pelvic pain and increased lower abdominal girth

Blood within the Fallopian tube is termed: A. Hydrosalpinx B. Hematosalpinx C. Pyosalpinx D. Hemosalpinx

B. Hematosalpinx

The malignant ovarian tumor with gastrointestinal origin is the: A. Brenner tumor B. Krukenberg tumor C. Yolk sac tumor D. Granulosa cell tumor

B. Krukenberg tumor

The ovary is supplied blood by the: a. Ovarian artery b. Ovarian artery and uterine artery c. Uterine artery d. Arcuate artery

B. Ovarian artery and uterine artery

All of the following adnexal masses may appear monographically similar to a uterine leiomyoma except: A. Thecoma B. Paraovarian cyst C. Fibroma D. Granulosa cell tumor

B. Paraovarian cyst

Pelvic ligament that extends from the lateral aspect of the uterus to the side walls of the pelvis

Broad ligament

What ovarian mass is associated with virilization? A. Krukenburg tumor B. Cystic teratoma C. Serous cystadenoma D. Sertoli-Leydig cell tumor

D. Sertoli-Leydig cell tumor

The pelvic ligament that provides support to the ovary to the pelvic side wall is the: a. Cardinal ligament b. Ovarian ligament c. Broad ligament d. Suspensory ligament of the ovary

D. Suspensory ligament of the ovary

What does the cumulus oophorus appear like?

Daughter cyst

The consequence of obstructions for congenital malformation of the vagina could lead to what?

Distention of the vagina, cervix, uterus and Fallopian tubes with blood or fluid

Dysfunctional uterine bleeding is usually related to what?

Endocrine abnormalities in which hormonal imbalances result in endometrial changes and subsequent abnormal bleeding... fibroid tumors, adenomyosis, endometrial hyperplasia, endometrial polyps, endometrial carcinoma

What are the potential differential gynecologic diagnosis of right upper quadrant pain?

Fitz-hugh-curtis syndrome

The name for the dominant follicle prior to ovulation

Graafian follicle

Excessive vomiting

Hyperemesis

The surgical removal of the uterus

Hysterectomy

Tube, the segment of the fallopian tube that is located between the interstitial and ampulla, uterus area of the uterus between the Corpus and the cervix

Isthmus

What happens to the dominant follicle during the luteal phase?

It becomes the corpus luteum, produces progesterone, preparing the endometrium for implantation.

The *hormone that surges* at ovulation is:

LH

What are the sonography findings of theca lutein cyst?

Large bilateral multiloculated ovarian cystic masses, may contain hemorrhagic components

The *periovulatory phase may also be referred* to as the:

Late proliferative phase

Fibromas are a benign ovarian mass that may be Complicated by a condition known as what?

Meigs syndrome

The first menstrual cycle

Menarche

Which of the following could also be described as *intermenstrual bleeding*?

Metrorrhagia

The surgical removal of a Myoma of the uterus

Myomectomy

Hydrocolpos is often seen in what age of patient?

Neonatal

What ligament extends from the ovary to the lateral surface of the uterus and supports ovaries?

Ovarian ligaments

What are the potential differential gynecologic diagnosis of elevated serum Alpha fetoprotein?

Ovarian yolk sac

An infection of the female genital tract that may involve the ovaries, uterus, fallopian tubes

Pelvic inflammatory disease

What are the clinical findings of vaginal obstructions?

Pelvic pain, enlarged uterus, abdominal pain, urinary retention, amenorrhea

What are the clinical findings of a mucinous cystadenoma?

Pelvic pressure and swelling

Another name for the late proliferative phase of the endometrial cycle which occurs around the time of ovulation

Periovulatory phase

What muscles can be confused with the ovaries or adnexal Mass?

Piriformis muscle or iliopsoas

Paired pelvic muscles located posteriorly that extends from the sacrum to the femoral greater trochanter

Piriformis muscles

The abnormal accumulation of fluid in the pleural space

Pleural effusion

Malignant sex cord stromal ovarian neoplasm that is associated with virilization

Sertoli leydig cell tumor

What are the sonography findings of a paraovarian cyst?

Simple cyst located adjacent been out of touch to the ovary, if hemorrhagic will appear complex

Each fallopian tube attaches to the uterus at the level of what?

Uterine horns called the cornua

What is the second most common malignant germ cell tumor?

Yolk Sac tumor

The arteries that directly supply blood to the functional layer of the endometrium are the: a. Radial arteries b. Spiral arteries c. Straight arteries d. Arcuate arteries

b. spiral arteries

The pelvic ligament that extends from the lateral aspect of the uterus to the side walls of the pelvis is the: a. Broad ligament b. Ovarian ligament c. Piriformis ligament d. Round ligament

broad ligament

the inferior portion of the cervix closest to the vagina is the: a. cornu b. internal os c. external os d. inferior fornix

c. external os

the superior portion of the cervix is the: a. cornu b. corpus c. internal os d. external os

c. internal os

which of the following would be the least likely to cause abdominal distension? a. ascites b. multiple leiomyoma c. ovarian hyperstimulation syndrome d. polycystic ovarian disease

!!! d. polycystic ovarian disease

What day is the menstrual cycle correlate with Menses at which time the endometrium is shed?

1 through 5

In secretory phase what is the endometrial thickness?

7 through 14 mm

The most common benign ovarian tumor is the: A. Cystic teratoma B. Mucinous cystadenoma C. Fibroma D. Sertoli- Leydig cell tumor

A. Cystic teratoma

The ovarian mass that contains fat, sebum, and teeth is the: A. Dermoid B. Fibroma C. Mucinous cystadenoma D. Yolk sac tumor

A. Dermoid

The ovarian tumor associated with an elevated serum lactate dehydrogenase is the: A. Dysgerminoma B. Sertoli-Leydig cell tumor C. Androblastoma D. Mucinous cystadenocarcinoma

A. Dysgerminoma

Which of the following is associated with the "whirlpool sign"? A. Ovarian torsion B. Hydrosalpinx C. Ovarian hyperstimulation syndrome D. Ovarian carcinoma

A. Ovarian torsion

A change in menstrual bleeding patterns due to either endocrine abnormalities or lesions within the uterus

Abnormal uterine bleeding

Cancer of the fallopian tube is rare and typically in the form of what?

Adenocarcinoma

The benign invasion of endometrial tissue into the myometrium of the uterus

Adenomyosis

What are the potential differential gynecologic diagnosis of menometrorrhagia?

Adenomyosis, endometrial polyp, pelvic inflammatory disease, perforated intrauterine device

What are the potential differential gynecologic diagnosis of dysmenorrhea?

Adenomyosis, endometriosis

What is the potential differential gynecologic diagnosis for chronic pelvic pain?

Adenomyosis, endometriosis, leiomyoma, pelvic inflammatory disease

What are the potential differential gynecologic diagnosis of dyspareunia?

Adenomyosis, endometriosis, pelvic inflammatory disease

What are the sonography findings of a Gartner duct cyst?

Anechoic Mass within the vagina

Normal ovarian flow is said to be: A. Low resistant during menstruation and high resistant during the proliferative phase B. High resistant during menstruation and low resistant at the time of ovulation C. Low resistant D. High resistant

B. High resistant during menstruation and low resistant at the time of ovulation

Prolapse of the pelvic organs most often involves the: a. Rectus abdominis and obturator internus muscles b. Levator ani and coccygeus muscles c. Obturator internus and levator ani muscles d. Piriformis and iliopsoas muscles

B. Levator ani and coccygeus muscles

The inner layer of the wall of the Fallopian tube is the: A.Muscular layer B. Mucosal layer C. Myometrial layer D. Serosal layer

B. Mucosal layer

The most common uterine anomaly in which the endometrium divides into two horns AKA bicornis unicollis

Bicornuate uterus

The stage at which the conceptus implants within the decidualized endometrium

Blastocyst

Is the involvement of adenomyosis focal or diffuse?

Both, more often within the posterior portion of the uterus

Physical defects that are present in a person at Birth

Congenital malformation

The peripheral arteries of the uterus are the: a. Radial arteries b. Spiral arteries c. Straight arteries d. Arcuate arteries

D) Arcuate arteries

What is the most common malignant germ cell tumor of the ovary?

Dysgerminoma

With menarche what may be influenced?

Environment and diet

Ovarian cyst that forms from the failure of the graafian follicle to ovulate

Follicular cyst

Blood accumulation within the uterine cavity

Hematometra

Blood accumulation within the uterus and vagina

Hematometrocolpos

A cyst that contains blood

Hemorrhagic cyst

What is the most common location for fibroids?

Intramural

What does the term metrorrhagia refer to?

Irregular menstrual bleeding between periods

The innominate bones of the pelvis consist of the: a. Ischium, ilium, and pubic bones b. Ilium, sacrum, and coccyx bones c. Sacrum, coccyx, and pubic bones d. Sacrum, ischium, and ilium bones

Ischium, ilium, and pubic bones

What happens to the corpus luteum if a pregnancy occurs?

It can reach a size of up to 10 cm but should resolve by 16 weeks of gestation.

Destruction or breaking down

Lysis

Posterior cul-de-sac

Male: between the urinary bladder and rectum; also referred to as the rectovesical pouch. Female: between the uterus and rectum; also referred to as pouch of Douglas and rectouterine pouch

What are the clinical findings of theca lutein cyst?

Markedly elevated levels of human chorionic gonadotropin as seen in cases of gestational trophoblastic disease, ovarian hyperstimulation, twin just stations,, nausea and vomiting, pain associated with hemorrhagic, rupture, and ovarian torsion

What are the clinical findings of a thecoma?

May be asymptomatic, postmenopausal vaginal bleeding or abnormal vaginal bleeding secondary to estrogen stimulation, make syndrome which is ascites and pleural effusion

may complicate fibroma with ascites and pleural effusion

Meig Syndrome

Ascites and pleural effusion in the presence of a benign ovarian tumor

Meigs syndrome

What is the definition of primary amenorrhea?

Menarche has not occurred by age 16 in a female patient.

Benign cyst located within the cervix

Nabothian cysts

A mass of tissue that contains abnormal cells AKA tumor

Neoplasm

What pelvic muscles are lateral to the ovaries?

Obturator internus

Pelvic ligament that provides support to the ovary extends from the ovary to the lateral surface of the uterus

Ovarian ligament

On Days 1 through 14 what phases are the ovary in endometrium in?

Ovary is in follicular and endometrium is in proliferative

What pelvic muscles are inferior near the vagina in transverse?

Pelvic diaphragm which includes levator Ani, coccygeus muscle

Infection of the female genital tract that may involve the ovaries, uterus, and or the fallopian tubes

Pelvic inflammatory disease

The outer layer of the uterus AKA serosal layer

Perimetrium

A syndrome characterized by anovulatory cycles, fertility, hirsutism, amenorrhea, and obesity AKA Stein Leventhal syndrome

Polycystic ovarian syndrome

An intraperitoneal extension of mucin secreting cells that result from the rupture of a malignant mucinous ovarian tumor or possibly a malignant tumor of the appendix

Pseudomyxoma peritonei

Intraperitoneal extension of mucin secreting cells that results from the rupture of a malignant mucinous ovarian tumor or possibly a malignant tumor of the appendix

Pseudomyxoma peritonei

An oily substance secreted by the sebaceous glands

Sebum

The outermost layer of the uterus AKA parimetrium

Serosal layer

What is the most common malignancy of the ovary?

Serous cystadenocarcinoma

25-year-old patient presented with hirsutism and abnormal periods. Sonography demonstrated a solid, complex ovarian mass. What is the most likely diagnosis?

Sertoli-Leydig cell tumor

What are the sonography findings of corpus luteum cysts and corpus luteum of pregnancy?

Simple cyst appearance however corpus luteum cyst may have a sticker wall and may be difficult to differentiate from other solid and cystic adnexal masses Hemorrhagic cyst variable appearances including complex components or entirely academic depending on the amount of blood and stage of lysis

What are the sonography findings of sertoli leydig cell tumors?

Solid, hypoechoic ovarian Mass, complex or partially cystic mass

retropubic space

Space of Retzius; between bladder and symphysis pubis

Functional ovarian cysts that are found in the presence of elevated levels of human chorionic gonadotropin, AKA theca luteal cyst

Theca lutein cysts

What are the largest and least common of the functional cysts?

Theca lutein cysts

A periovulatory endometrial sonography appearance in which the outer echogenic basal layer surrounds the more hypoechoic functional layer while the functional layer is separated by the echogenic endometrial stripe

Three line sign

What is the sonographic appearance of the proliferative endometrium?

Three-line sign (echogenic basal layer - hypoechoic functional layer - echogenic canal - hypoechoic functional layer - echogenic basal layer)

Inferior portion of the pelvis that contains the uterus, ovaries, Fallopian tubes, urinary bladder, small bell, sigmoid colon, rectum

True pelvis

Inferior portion of the pelvis that contains the uterus, ovaries, Fallopian tubes, urinary bladder, small bowel, sigmoid colon, and rectum

True pelvis

Describe the condition of stage 2 of ovarian carcinoma

Tumor involves one or both ovaries with pelvic extension

What structure produces *hormones that directly act upon the endometrium* to produce varying thicknesses and sonographic appearances?

Uterus

Congenital malformation of the uterus that results in complete duplication of the uterus, cervix, vagina

Uterus didelphys

The uterine arteries Supply blood to where?

Uterus, Fallopian tubes, ovaries

What develop from the paired mullerian ducts?

Uterus, vagina, fallopian tubes

Yolk Sac tumor is often seen in whom?

Younger than 20 with high malignancy, poor prognosis

which of the following is typically not a clinical complaint of women who are suffering from adenomyosis? a. amenorrhea b. dysmenorrhea c. dyspareunia d. menometrorrhagia

a. amenorrhea (heavy periods)

the uterine position in which the corpus tilts forward and comes in contact with the cervix describes: a. anteflexion b. anteversion c. retroflexion d. retroversion

a. anteflexion

the largest part of the uterus is the: a. corpus b. isthmus c. cervix d. fundus

a. corpus

the surgical removal of a fibroid is termed: a. hysterosonogram b. total abdominal hysterectomy c. myomectomy d. uterine artery embolization

c. myomectomy

leiomyosarcoma of the uterus denotes: a. the benign invasion of endometrial tissue into the myometrium b. the ectopic location of endometrial tissue into the adnexa c. the malignant counterpart of a fibroid d. an anechoic, simple cyst located within the cervix

c. the malignant counterpart of a fibroid

congenital malformation of the uterus that results in complete duplication of the genital tract is: a. unicornuate uterus b. bicornis bicollis c. uterus didelphys d. subseptate uterus

c. uterus didelphys

when does the Centers for Disease Control recommend that alcoho-based handrub not be used by the sonographer?

c. when your hands are visibly soiled

after ovulation, what forms on the ovary and produces progesterone

corpus luteum

develops after ovulation from ruptured follicle, produces progesterone maintaining endo

corpus luteum cyst

50 to 70% benign asymptomatic large and bilateral anechoic lesion with septations or papillary projections

cystadenoma

A drug administered to pregnant women from the 1940s to the 1970s to treat threatened abortions and premature labor that has been linked with uterine malformation in the exposed fetus

do ethyl stop best too DES

what happens when ovulation occurs

dominant follicle ruptures releasing ovum and follicular fluid

first half of menstrual cycle

follicular phase

fingerlike projections form infundibulum

frimbria

The paired muscles that are located lateral to the uterus and anterior to the iliac crest are the: a. Iliopsoas muscles b. Rectus abdominis muscles c. Obturator interni muscles d. Piriformis muscles

iliopsoas muscles

primary purpose of fallopian tubes

provide area of fertilization and to offer a means of transportation for product of conceptus

Which vessels supply blood to the deeper layers of the myometrium? a. Radial arteries b. Spiral arteries c. Straight arteries d. Arcuate arteries

radial arteries

What are the potential differential gynecologic diagnosis of urinary frequency?

Leiomyoma, leiomyosarcoma

During pregnancy the isthmus may be referred to as what?

Lower uterine segment

The total number of completed pregnancies that I've reached the age of viability

Parity

Something that grows off a stalk

Pedunculated

Contractions that move in a wave-like pattern to propel a substance

Peristalsis

Location of leiomyoma in which the tumor grows outward and distorts the Contour of the uterus

Subserosal

Who recommends the proper staging of ovarian carcinoma?

The international Federation of Gynecology and obstetrics FIGO

Following Menses how does the endometrium appear?

Thin package anticline

The boundaries of the female pelvis are considered to be from what?

Iliac crest to a group of muscles known as the pelvic diaphragm located at the base of the pelvis

What pelvic muscles are lateral and anterior to iliac crest?

Iliopsoas

False pelvis bilateral muscles located lateral to the uterus and anterior to the iliac crest

Iliopsoas muscle

The innominate bones consist of what?

Ilium, ischium, pubic symphysis

A vaginal anomaly in which the hymen has no opening, therefore resulting in an obstruction of the vagina

Imperforate hymen

Hypothalamus a vaginal anomaly in which the hymen has no opening resulting in an obstruction of the vagina

Imperforate hymen

How might a Cervical Carcinoma appear sonography?

In homogeneous, enlarged cervix or a focal Mass within the cervix

The distal segment of the fallopian tube

Infundibulum

What are the sonography findings of pyosalpinx and hematosalpinx?

Internal components and may appear echogenic or have a fluid fluid level

Internal branches of the common iliac arteries

Internal iliac arteries

The right and left uterine arteries are branches of what?

Internal iliac arteries

The superior portion of the cervix closest to the isthmus

Internal os

What hormone gives the surge to trigger ovulation

LH

The *two hormones produced by the anterior pituitary gland* that impact the menstrual cycle are:

LH and FSH

During which phase of the endometrial cycle would the endometrium yield the *three-line sign*?

Late proliferative

What are the potential differential gynecologic diagnosis of palpable adnexal Mass?

Leiomyoma, ovarian Mass, pelvic inflammatory disease

An elevated white blood cell count

Leukocytosis

Imaginary line that separates the true pelvis from the false pelvis

Linea terminalis

Menstrual bleeding

Menses

Fibromas are most often found in what aged women?

Middle-aged

Are ovarian fibromas associated with estrogen?

No

Days 15 through 28 what phase is over in endometrium in?

Ovary is in luteal and endometrium is in secretory

Leiomyosarcoma are more commonly found in what woman?

Perimenopausal or postmenopausal

Inflammation of the peritoneal lining

Peritonitis

What pelvic muscles are posterior?

Piriformis

Thecoma are most often found in whom?

Postmenopausal women, Meigs syndrome

Paired anterior abdominal muscles that extend from the xiphoid process of the sternum to the pubic bone

Rectus abdominis muscles

What age are dermoids commonly found in?

Reproductive age group but may also be found in the postmenopausal patients

Inflammation of the fallopian tubes

Salpingitis

Ovarian tumors that arise from The gonadal Ridges

Sex cord stromal tumors

What are the sonography findings of granulosa cell tumor?

Solid hypoechoic Mass, complex or partially cystic mass

Coiled arteries that Supply blood to the functional layer of the endometrium

Spiral arteries

Uterine radial artery branch that supplies blood to the basal layer of the endometrium

Straight arteries

Estrogen is produced throughout the menstrual cycle it is initially produced by what?

Theca internal cells of the secondary follicles during the first part of the menstrual cycle, during this phase estrogen initiates the proliferation and thickness of the endometrium by encouraging the growth and expansion of spiral arteries and glands within the functional layer of the endometrium, oxygen has many other important functions, regeneration of the endometrium after Menses and induction of salt and water retention, simulates contractile emotions within the uterine myometrium and Fallopian tubes, during second half dimensional cycle following ovulation progesterone is produced by the corpus luteum of the ovary, progesterone is responsible for maintaining the thickness of the endometrium and inducing its secretory activity as it is prepared for implantation

Benign ovarian sex cord stromal tumor that produces estrogen in older woman

Thecoma

In the secretory phase how does endometrium appear?

Thickened and academic measuring between 7 and 14 mm

The ovaries form and where and descend to the pelvis in utero?

Upper abdomen

What are the sonography findings of yolk Sac tumor?

Varying sonography appearances

Changes within the female that are caused by increased androgens, may lead to deepening of the voice in hirsutism

Virilization

Where is the ovum contained?

Within the cumulus oophorus

What term describes the echogenicity of a simple ovarian cyst?

a. anechoic

abnormally heavy and prolonged menstrual flow between periods is termed: a. menometrorrhagia b. menarche c. menorrhagia d. dysmenorrhea

a. menometrorrhagia

all of the following are clinical findings associated with leiomyoma except: a. myometrial cyst b. infertility c. palpable pelvic mass d. menorrhagia

a. myometrial cyst

precocious puberty is defined as the development of pubic hair, breasts, and the genitals before the age of: a. 13 b. 8 c. 5 d. 10

b. 8

what Doppler artifact occurs when the Doppler sampling rate is not high enough to display the Doppler shift frequency?

b. aliasing

the inner mucosal lining of the uterus is the: a. myometrium b. endometrium c. serosal layer d. perimetrium

b. endometrium

Anechoic fluid noted distending the uterus and cervix within a pediatric patient is termed: a. hydrocolpos b. hydrometrocolpos c. hydrometra d. hematometrocolpos

b. hydrometrocolpos

having the same echogenicity means:

b. isoechoic

what section of the uterus is also referred to as the lower uterine segment: a. cervix b. isthmus c. fundus d. cornu

b. isthmus

- small, solid, hypoechoic, unilateral tumors with calcifications - benign

brenner tumor aka transitional cell tumor

amernorrhea is defined as:

c. lack of menstrual flow

what leiomyoma location have an increased risk to undergo torsion? a. subserosal b. intracavity c. pedunulated d. submucosal

c. pedunulated

hairlike structures in the tube lumen

cilia shift

- considered most common cyst of 1st trimester pregnancy - often complex, thick walled - may resemble ectopic pregnancy

corpus luteum cyst

leiomyomas that project from a stalk are termed: a. submucosal b. intramural c. subserosal d. pedunculated

d. pedunculated

which of the following would be considered the more common uterine anomaly? a. bicornis univernus b. bicornus bicollis c. uterus didelphys d. septate uterus

d. septate uterus

if fertilization occurs menses begins when

day 1 of cycle

FSH causes what

follicle maturation

The hormone produced by the *trophoblastic cells* of the early placenta is:

hCG

the ovum is contained where

in the cumulus oophorus of dominant follicle

Hammock shaped pelvis muscle group located between the coccyx and pubis consisting of iliococcygeus, pubococcygeus and puborectalis

levator ani muscles

The pelvic muscle group that is located between the coccyx and the pubis is the: a. Levator ani muscles b. Rectus abdominis muscles c. Obturator internus muscles d. Piriformis muscle

levator ani muscles

second half of menstrual cycle

luteal phase

late proliferative phase is also known as

periovulatory phase

Pelvic bones, when visualized on sonography, will produce: a. Posterior shadowing b. Posterior enhancement c. Mirror image artifact d. Minimal enhancement

posterior shadowing

the fluid from the ruptured follicle most often will settle in the ...

rectouterine pouch (pouch of douglas)

pelvic muscles that can be sonographically identified include

rectus abdominis, iliopsoas, obturator internus, piriformis, levator ani, coccygeus

may be associated with endocrinologic abnormalities

secondary amenorrhea

occurs after ovulation and stimulated by progesterone

secretory phase

the testicular equivalent of an ovarian dysgerminoma is the ...

seminoma

most common malignancy in the ovary, bilateral, thicker septations, more projections

serous cystadenoma

what two thing comprise majority of neoplasms of the ovary

serous cystadenomas and cystic teratomas

sonographic appearance of fallopian tube carcinoma is what

solid mass within the adnexa

what sign from CD may be present in a torsed ovary

whirlpool sign

As a result of constant estrogen stimulation postmenopausal patients with granulosa cell tumors have approximately what percentage chance of developing endometrial carcinoma?

10 to 15

*Ovulation* typically *occurs* on day-of the menstrual cycle.

14

around what day is there a surge in LH

14

The normal pre-pubertal uterus has a cervix to uterus ratio of what?

2 to 1

The fingerlike extensions of the Fallopian tube are called: A. Fimbria B. Infundibulum C. Cilia D. Ampulla

A. Fimbria

A 55-year old patient presents to the sonography department with a history of pelvic pressure, abdominal swelling, and abnormal uterine bleeding. A pelvic sonogram reveals a large, multiloculated cystic mass with papillary projections. What is the most likely diagnosis? A. Serous cystadenocarcinoma B. Cystic teratoma C. Androblastoma D. Dysgerminoma

A. Serous cystadenocarcinoma

A change in *menstrual bleeding associated with lesions* within the uterus relates to:

AUB

In the early proliferative phase the endometrium appears how?

Academic and then typically measuring 4 mm on day 4 and 8 mm on day 8 of the cycle

Congenital malformations of the vagina can lead to what?

Accumulation of fluid within the female genital tract secondary to an obstruction

What are the clinical findings of ovarian torsion?

Acute unilateral abdominal or pelvic pain, nausea and vomiting

What are the potential differential gynecologic diagnosis of enlarged uterus?

Adenomyosis, endometrial carcinoma, leiomyoma, leiomyosarcoma

The size and shape of the uterus depends on what?

Age of patient, parity, presence of pathology or congenital anomalies

The typical version of the uterus where the uterine body tilts forward, forming a 90 degree angle with the cervix

Anteversion

The normal position of the uterus is considered to be what?

Anteversion or anteflexion

Major abdominal artery responsible for supplying the abdomen, pelvis, and lower extremities with oxygenated blood

Aorta

What are the clinical findings of a nabothian cyst?

Asymptomatic

Degeneration of a follicle

Atresia or ovarian follicle

Pus within the Fallopian tube is termed: A. Hematosalpinx B. Pyosalpinx C. Hydrosalpinx D. Hemosalpinx

B. Pyosalpinx

Another name for the endometriomas

Chocolate cysts

Hairlike projections within the fallopian tube

Cilia

The space of Retzius is located: a. Between the uterus and bladder b. Between the bladder and ilium c. Along the lateral aspect of the uterus d. Between the bladder and pubic bone

D. Between the bladder and pubic bone

The ovarian cysts that are most often bilateral and are associated with markedly elevated levels of hCG are the: A. Corpus luteum cysts B. Paraovarian cysts C. Granulosa cell cysts D. Theca lutein cysts

D. Theca lutein cysts

A small cyst within a large cyst

Daughter cyst

What are the sonography findings of adenomyosis?

Diffusely enlarged uterus, hypoechoic or academic areas adjacent to the endometrium, heterogeneous myometrium, myometrial cyst, ill-defined interface between myometrium and endometrium, thickening of the posterior myometrium

In the neonatal period what does the uterus look like?

Distinct endometrial echoes in the first week of life

Painful or difficult urination

Dysuria

Theca lutein cysts are found in the presence of what? What are common concurrent conditions?

Elevated levels of human chorionic gonadotropin in excess of 100,000,, gestational trophoblastic disease and ovarian hyperstimulation syndrome and multiple just stations

What are the clinical findings of yolk Sac tumor?

Elevation in serum AFP

The degeneration of the endometrium with advancing age most often seen in postmenopausal women

Endometrial atrophy

Cancer of the endometrium

Endometrial carcinoma

An *increase in the number of endometrial cells* is termed:

Endometrial hyperplasia

An increase in the number of endometrial cells

Endometrial hyperplasia

What are the potential differential gynecologic diagnosis of intermenstrual bleeding?

Endometrial polyp

Small nodules of hyperplastic endometrial tissue

Endometrial polyps

Benign, blood containing tumor that forms from the implementation of ectopic endometrial tissue, tumor associated with endometriosis

Endometrioma

Functional ectopic endometrial tissue located outside the uterus

Endometriosis

The *hormone of the pituitary gland that stimulates* follicular development of the ovary is:

FSH

Superior portion of the pelvis

False pelvis

What treatment will also result in the development of multiple, enlarged follicular cysts?

Fertility treatment

Adenomyosis is often present in the uterus Afflicted with what?

Fibroid tumors

the dominant follicle

Graafian follicle

A laboratory value that indicates the amount of red blood cells in blood

Hematocrit

Blood within the fallopian tube

Hematosalpinx

The inability to conceive a child after one year of unprotected intercourse

Infertility

The segment of the fallopian tube that lies within the uterine horn

Interstitial

A leiomyoma located within the uterine

Intracavitary

What kind of glands are the ovaries?

Intraperitoneal and endocrine

Area of the uterus between the Corpus and the cervix

Isthmus

Malignant ovarian tumor that metastasizes from the GI tract

Krukenberg tumor

What ovarian neoplasm is a result of metastases of a gastrointestinal cancer?

Krukenberg tumor

formula for ovarian volume

LENGTH X WIDTH X HEIGHT X 0.523

what artifact could be noted emanating from air or gas within the endometrium in a patient with endometritis?

!!! a. Ring-down

endovaginal transducers may be cleaned by submerging in a(n) ____-based solution:

!!! a. glutaraldehyde

which of the following laboratory tests may be used as a tumor marker for an ovarian dysgerminoma?

!!! a. lactate dehydrogenase

Ovulation typically occurs around what day?

14

Follicular cysts range in size from what?

3 to 8 cm

How many layers does the fallopian tube have and what are they?

3, outer serosa, middle muscular layer, inner mucosal layer

How many stages of ovarian carcinoma are there?

4

In the early proliferative phase what is endometrial thickness?

4 through 8 mm

The *measurement of the endometrium during the early* proliferative phase ranges from:

4 to 8 mm

In Periovulatory phase what is endometrial thickness?

6 through 10 mm

Which of the following is also referred to as a chocolate cyst? A. Endometrioma B. Endometroid C. Cystic Teratoma D. Androblastoma

A. Endometrioma

A focal mass of adenomyosis

Adenomyoma

Shadowing ovarian torsion is much more common in whom?

Adolescence

Failure of an organ to grow during embryologic development

Agenesis

A protein produced by the fetal yolk Sac, fetal gastrointestinal tract, and the fetal liver may also be produced by some malignant tumors

Alpha fetoprotein

Absence of menstrual period

Amenorrhea

The absence of menstrual period

Amenorrhea

The absence of menstrual.

Amenorrhea

The longest and most tortuous segment of the fallopian tube

Ampulla

*Ectopic endometrial tissue* within the uterus that *leads to AUB* is termed:

Andenomyosis

Sertoli leydig cell tumor AKA

Androblastoma

Peritoneal outpouching located between the bladder in the uterus AKA vesico uterine pouch

Anterior cul-de-sac

Peritoneal outpouching located between the bladder in the uterus AKA vesicouterine pouch

Anterior cul-de-sac

What are the potential differential gynecologic diagnosis of infertility?

Asherman syndrome, endometrial carcinoma, endometrial polyp, endometriosis, leiomyoma, pelvic inflammatory disease, polycystic ovarian disease

What are the potential differential gynecologic diagnosis of post dilation and postcurettage?

Asherman syndrome, endometritis, retained products of conception

What are the clinical findings of a fibroma?

Asymptomatic, Meigs syndrome

What are the clinical findings of a Brenner tumor?

Asymptomatic, Meigs syndrome, palpable mass or pain

The longest and most tortuous segment of the Fallopian tube is the: A. Fimbria B. Ampulla C. Isthmus D. Interstitial

B. Ampulla

What substance does hysterosalpingography utilize for the visualization of the uterine cavity and Fallopian tubes? A. Saline B. Radiographic contrast C. Water D. Betadine

B. Radiographic contrast

Which of the following is the most common malignancy of the ovary? A. Cystic teratoma B. Serous cystadenocarcinoma C. Krukenberg tumor D. Sertoli- Lyedig cell tumor

B. Serous cystadenocarcinoma

The ovarian cyst associated with gestational trophoblastic disease is the: A. Corpus luteum cyst B. Theca lutein cyst C. Dermoid cyst D. Paraovarian cyst

B. Theca lutein cyst

The non-functional outer layer of the endometrium

Basal layer

The follicular phase of the ovarian cycle is considered to begin on what day and last until what day?

Begins on day one and lasts until day 14 thus in effect with ovulation

What ligament extends from the lateral aspect of the uterus to the side walls of the pelvis and supports the uterus, tubes, ovaries?

Broad ligaments

The muscle located lateral to the ovaries is the: a. Iliopsoas muscle b. Rectus abdominis muscle c. Obturator internus muscle d. Piriformis muscle

C) Obturator internus muscle

Pelvic muscles appear: a. Echogenic b. Anechoic c. Hypoechoic d. Complex

C) hypoechoic

Hairlike projections within the fallopian tube are called: A. Interstita B. Fimbria C. Cilia D. Peristalsis

C. Cilia

After the Graafian follicle ruptures, the remaining structure is termed the: A. Graafian remnant B. Corpus albicans C. Corpus luteum D. Theca lutein cyst

C. Corpus luteum

Sonographically, which of the following would most likely be confused for a pedunculated fibroid tumor because of its solid appearing structure? A. Serous cystadenoma B. Mucinous cystadeoma C. Fibroma D. Theca lutein cyst

C. Fibroma

With what ovarian tumor is Meigs syndrome most likely associated? A. Dysgerminoma B. Cystic teratoma C. Fibroma D. Yolk sac tumor

C. Fibroma

The right ovarian vein drains directly into the: a. Right renal vein b. Aorta c. Inferior vena cava d. Common iliac vein

C. Inferior vena cava

The true pelvis is delineated from the false pelvis by the: a. Space of Retzius b. Adnexa c. Linea terminalis d. Iliac crest

C. Linea terminalis

An endometrioma most likely appears as a: A. Simple, anechoic mass with through transmission B. Complex mass with internal shadowing components C. Mostly cystic mass with low-level echoes D. Solid, hyper echoic shadowing mass

C. Mostly cystic mass with low-level echoes

What would be a predisposing condition that would increase the risk for suffering from ovarian torsion? A. Hirsutism B. Excessive exercise C. Ovarian mass D. Sonohysterography

C. Ovarian mass

elevated levels of what is a protein that is increased in women with ovarian cancer

CA- 125

A diagnostic modality that utilizes ionizing radiation to produce images of the human body in cross-sectional and reconstructed 3D formats

Computed tomography

Primary amenorrhea may be caused by what?

Congenital abnormalities or congenital obstruction such as imperforate hymen

The remaining structure of the corpus luteum after it is deterioration

Corpus albicans

The remaining structure of the corpus luteum after its deterioration

Corpus albicans

Which structure remains after the *corpus luteum has regressed*?

Corpus albicans

After the graafian follicle is ruptured its structure is converted into what?

Corpus luteum

Temporary endocrine gland that results from the rupture of the graafian follicle after ovulation

Corpus luteum

The *temporary endocrine gland* that results from the rupture of the Graafian follicle is the:

Corpus luteum

The temporary endocrine gland that results from the rupture of the graafian follicle after ovulation

Corpus luteum

physiologic cyst that develops after ovulation has occurred

Corpus luteum cyst

Monthly symptoms of menstruation without bleeding

Cryptomenorrhea

Structure that contains the developing oocyte

Cumulus oophorus

The structure noted within the *Graafian follicle containing the developing ovum* is the:

Cumulus oophorus

The structure that contains the developing oocyte

Cumulus oophorus

Benign ovarian mass that is composed of the three germ cell layers AKA dermoid cyst

Cystic teratoma

The bilateral muscles that are located posterior to and extend from the sacrum to the femoral greater trochanter are the: a. Levator ani muscles b. Rectus abdominis muscles c. Obturator internus muscles d. Piriformis muscles

D) Piriformis muscles

The segment of Fallopian tube where fertilization typically occurs is the: A. Cornu B. Fimbria C. Interstitial D. Ampulla

D. Ampulla

Which of the following is a tumor of ectopic endometrial tissue? A. Brenner tumor B. Cystic teratoma C. Yolk-sac tumor D. Endometrioma

D. Endometrioma

The most distal part of the Fallopian tube is the: A. Cornu B. Ampulla C. Interstitial D. Infundibulum

D. Infundibulum

The short and narrow segment of the Fallopian tube distal to the interstitial segment is the: A. Ampulla B. Fimbria C. Infundibulum D. Isthmus

D. Isthmus

The malignant ovarian mass that is associated with pseudomyxoma peritonei is the: A. Dysgerminoma B. Sertoli-Leidig cell tumor C. Serous cystadenocarcinoma D. Mucinous cystandenocarcinoma

D. Mucinous cystandenocarcinoma

Another name for a cystic teratoma

Dermoid cyst

Mass of hair within a cystic teratoma

Dermoid mesh

Part of a dermoid tumor that contains various tissues and may produce the posterior shadowing during a sonography exam

Dermoid plug

Describe the condition of stage 4 ovarian carcinoma

Distant Mets behind the peritoneal cavity

What are the sonography findings of vaginal obstructions?

Distention of the uterus, vagina, or both with anechoic or complex fluid

Abnormal uterine bleeding is usually caused by what?

Endocrine abnormalities or lesions within the uterus

Area that lies between the two layers of the endometrium AKA uterine cavity

Endometrial cavity

What are the potential differential gynecologic diagnosis of tamoxifen therapy?

Endometrial hyperplasia

What is the most likely explanation for abnormal uterine bleeding with an endometrial measurement of <5 mm in a postmenopausal patient?

Endometrial hyperplasia

What are the potential differential gynecologic diagnosis of menorrhagia?

Endometrial hyperplasia, endometriosis, leiomyoma, leiomyosarcoma

An important differentiation should be made between endometriosis and adenomyosis. Discuss the differences.

Endometriosis tend to be younger and have fertility problems while adenomyosis are often older and multiparous

The hormone released by the ovary during the proliferative phase then initiates the proliferation and thickening of the endometrium

Estrogen

Which hormone released by the ovary during proliferative phase *stimulates endometrial thickening*?

Estrogen

The ovary produces what two hormones during the menstrual cycle?

Estrogen and progesterone

What are the potential differential gynecologic diagnosis of lost intrauterine contraceptive device?

Expelled intrauterine device, perforated myometrium / uterus

The inferior portion of the cervix in close contact with the vagina

External OS

External branches of the common iliac arteries

External iliac arteries

The vagina is a tubular organ which extends from where to where?

External os of the cervix to the external genitalia

During the follicular phase the anterior pituitary gland secretes what which initiates the follicular development of the ovary many follicles are produced by the ovary?

FSH

during the follicular phase, the ant pit gland produces what

FSH

two main hormones produced by the anterior pituitary gland

FSH and LH

ovaries are stimulated by what

FSH produced by anterior pituitary gland

A patient presents with abnormal uterine bleeding. What is the most common cause?

Fibroid tumors invading the endometrial cavity

An ovarian sex cord stromal tumor found in middle aged women

Fibroma

The finger-like extensions of the fallopian tube located on the infundibulum

Fimbria

The first day of the menstrual cycle is said to occur on what?

First day of bleeding

Small round group of cells

Follicle

Should the graafian follicle failed to ovulate it could continue to enlarge and result in what?

Follicular cyst

The *first phase of the ovarian cycle* is the:

Follicular phase

The first phase of ovarian cycle

Follicular phase

The ovarian cycle consists of what two phases?

Follicular phase and luteal phase

The most Superior and widest portion of the uterus

Fundus

The uterus can be divided into what four major divisions?

Fundus, Corpus, Isthmus, cervix

I type of neoplasm derived from germ cells of the gonads, may also be found outside of the reproductive tract

Germ cell tumor

A disease associated with an abnormal proliferation of the trophoblast cells during pregnancy AKA molar pregnancy

Gestational trophoblastic disease

Blood accumulation within the vagina

Hematocolpos

What are the sonography findings of uterine leiomyoma?

Hypoechoic Mass within the uterus, posterior shadowing from the mass, degenerating fibroids may have calcifications or cystic components, multiple fibroids appear as an enlarged irregular shaped diffusely heterogeneous uterus

What are the sonography findings of a thecoma?

Hypoechoic solid Mass with posterior attenuation, no posterior enhancement, if large May mimic a pedunculated leiomyoma

What are the potential differential gynecologic diagnosis of virilization?

Sertoli leydig cell tumor, ovarian carcinoma

The cervix is marked Superiorly by what and inferiorly by what?

Superior is the internal os and inferior is the external os

What are the clinical findings of serous and mucinous cystadenocarcinoma?

Weight loss, pelvic pressure and swelling, abnormal vaginal bleeding, GI symptoms, acute abdominal pain associated with torsion or rupture, elevated ca-125

Fluid noted posterior to the uterus would most likely be located within the: a. Space of Retzius b. Pouch of Douglas c. Anterior cul-de-sac d. Adnexa

b. pouch of douglas

common suspect of AUB

fibroid tumors, leiomyomas

At what day of the menstrual cycle does ovulation occur as the dominant follicle ruptures releasing the mature ovum and a small amount of follicular fluid into the peritoneal cavity?

14

fertilization occurs on what day

15

Excess fluid in the peritoneal cavity

Ascites

What are the potential differential gynecologic diagnosis of Meigs syndrome?

Brenner tumor fibroma thecoma

Small benign ovarian tumors

Brenner tumors

What is the most common female malignancy in women younger than 50?

Cervical Carcinoma

The rigid region of the uterus located between the Isthmus and vagina

Cervix

What are the clinical findings of dysgerminoma?

Children - precocious puberty, elevated serum lactate dehydrogenase, possible elevated serum HCG

Abdominal aortic bifurcation vessels

Common iliac arteries

What is the largest part of the uterus?

Corpus or body

*FSH is produced* by the:

Anterior pituitary gland

*LH is produced* by the:

Anterior pituitary gland

The anterior segment of the pituitary gland which is responsible for releasing follicle-stimulating hormone and luteinizing hormone during the menstrual cycle

Anterior pituitary gland

Where can endometriomas be located?

Anywhere outside of the endometrial cavity including on any other pelvic organs such as the bladder and bowel but are more commonly found in the ovary

Peripheral arteries of the uterus that lie at the edge of the myometrium

Arcuate arteries

What are the branches of the uterine artery?

Arcuate artery then to radial arteries then to straight arteries and spiral

A collection of abdominal fluid within the peritoneal cavity

Ascites

What are the potential differential gynecologic diagnosis of abdominal distention?

Ascites, leiomyoma, ovarian hyperstimulation syndrome, ovarian malignancy

What are the potential differential gynecologic diagnosis of hypomenorrhea?

Asherman syndrome

What is the potential differential gynecologic diagnosis for amenorrhea?

Asherman syndrome, polycystic ovarian disease

What are the clinical findings of a Gartner duct cyst?

Asymptomatic

What are the clinical findings of serous cystadenoma?

Asymptomatic

What are the clinical findings of a follicular cyst?

Asymptomatic, pain associated with Hemorrhage and enlargement of cyst

Painful sexual intercourse

Dyspareunia

A pregnancy located outside of the endometrial cavity of the uterus

Ectopic pregnancy

Hematocrit is laboratory value that should be evaluated in cases of suspected what?

Ectopic pregnancy

Secondary amenorrhea may be associated with what?

Endocrinologic abnormalities or pregnancy, secondary amenorrhea that is not associated with pregnancy is characteristically diagnosed and the postmenarcheal woman who has had at least 12 months without a menstrual cycle

Yolk Sac tumor AKA?

Endodermal sinus tumor

A catheter placed into the urinary bladder via the urethra that is used to drain urine, it can also be claimed and used to temporarily distend the bladder for a pelvic sonography

Foley catheter

Endovaginal transducers should be soaked in what?

Glutaraldehyde based solution

The *hormone produced by the hypothalamus* that *controls* the release of the hormones for menstruation by the anterior pituitary gland is:

GnRH

The *dominant follicle* prior to ovulation is termed the:

Graafian follicle

How is the volume of the ovary determined?

Length X width X height X 0.5233.2

The hormone of the anterior pituitary gland that surges around day 14 of the menstrual cycle resulting in ovulation

Luteinizing hormone

What are the sonography findings of a krukenberg tumor?

Bilateral, smooth walled hypoechoic ovarian Mass, may have ascites

The pituitary gland AKA? Located where? Responsible for?

Master gland, endocrine gland located within the brain that consists of an anterior and posterior lobe, anterior lobe responsible for the release of two hormones follicle-stimulating hormone and luteinizing hormone, FSH causes the development of multiple follicles in the ovaries, LH surges around day 14 of the menstrual cycle which results in ovulation

A diagnostic imaging modality that utilizes the administration of radionuclides into the human body for an analysis of the function of organs, or for the treatment of various abnormalities

Nuclear medicine

What are the clinical findings of a cystic teratoma?

Often asymptomatic, if torsion or rupture occurs the patient may present with acute pelvic pain

What are the ovaries consist of?

Outer cortex and inner medulla. Medulla contains ovarian vasculature and lymphocytes, cortex involves the mass of the ovary and is the site of the oogenesis

The suspensory ligament of the ovary contains what?

Ovarian artery, ovarian vein, lymphatics, ovarian nerves

The surgical removal of an ovarian cyst

Ovarian cystectomy

What are the potential differential gynecologic diagnosis of elevated serum human chorionic gonadotropin in non gravid?

Ovarian dysgerminoma

Which of the following would *not be a cause of AUB (abnormal uterine bleeding)*?

Ovarian torsion

Following Menses the ovary is in what phase while the endometrium is in what phase?

Ovary is in the follicular phase while the endometrium is in the proliferative phase

What are the sonography findings of dysgerminoma?

Ovoid, solid echogenic Mass on the ovary, may contain some cystic components

A tumor marker in the blood that can indicate certain types of cancer such as cancer of the ovary, endometrium, breast, gastrointestinal tract, and lungs, stands for cancer antigen 125

CA - 125

The anterior cul-de-sac is also referred to as the: a. Space of Retzius b. Rectouterine pouch c. Pouch of Douglas d. Vesicouterine pouch

D. Vesicouterine pouch

What are the sonography appearance of a mucinous cystadenoma?

Large predominantly anechoic lesion that contains septations and or papillary projections, may contain some recognizable internal debris, not often bilateral

What are the sonography findings of serous cystadenoma?

Large, bilateral, predominantly anechoic lesion that contain septations and or papillary projections

Paired pelvic muscles located lateral to the ovaries

Obturator internus muscle

An abnormality that results from the ovary twisting on its mesenteric connection, consequently cutting off the blood supply to the ovary

Ovarian torsion

Together what comprise most neoplasms of the ovary?

Serous cystadenoma and cystic teratoma

upon sonographic evaluation of a patient complaining of abnormal distention, you visualize a large, hypoechoic mass distorting the anterior border of the uterus. what is the most likely location of the mass? a. intramural b. subserosal c. submucosal d. intracavity pedunculated

b. subserosal

pseudoprecocious puberty may be associated with all of the following except: a. ovarian tumor b. adrenal tumor c. liver tumor d. brain tumor

d. brain tumor

paracolic gutters

extend alongside the ascending and descending colon on both sides of the abdomen

may be referred to as oviducts, uterine tubes, or saplings

fallopian tubes

- sex cord stromal not associated with estrogen production - benign masses - middle aged women - hypoechoic, solid, post attenuation

fibroma

What is an estrogen producing tumor that is considered to be the most common estrogenic tumor?

Granulosa cell tumor

What are some laboratory values that may warrant a pelvic sonogram?

Human chorionic gonadotropin, hematocrit, white blood cell count

What is the typical sonographic *appearance of the endometrium during the secretory phase*?

Hyperchoic and thick

What is treatment for adenomyosis?

Hysterectomy or hormone therapy

What are the potential differential gynecologic diagnosis of elevated ca-125?

Endometriosis, leiomyoma, ovarian carcinoma, pelvic inflammatory disease

What are the potential differential gynecologic diagnosis of fever?

Endometritis, pelvic inflammatory disease

A means of looking inside the human body by utilizing an endoscope

Endoscopy

Excessive hair growth in women in areas where hair growth is normally negligible

Hirsutism

The dilation of the renal collecting system resulting from the obstruction of the flow of urine from the kidneys to the bladder AKA pelvocaliectasis

Hydronephrosis

Decreased or scant menstrual flow

Hypomenorrhagia

A common form of birth control in which a small device is placed within the endometrium to prevent the implementation of a pregnancy AKA intrauterine contraceptive device

Intrauterine device

What are the potential differential gynecologic diagnosis of dysuria?

Leiomyoma, leiomyosarcoma

A diagnostic modality that utilizes electromagnetic radiation to produce images of the human body in cross-sectional and reconstructed 3D formats

Magnetic resonance imaging

Heavy and prolonged bleeding between periods

Menometrorrhagia

What are the potential differential gynecologic diagnosis of acute pelvic pain?

Ovarian mucinous cystadenocarcinoma, ovarian mucinous cystadenoma, ovarian serous cystadenocarcinoma, ovarian serous cystadenoma, ovarian torsion, pelvic inflammatory disease, ruptured ovarian hemorrhagic cyst, perforated intrauterine contraceptive device

which of the following is best defined as difficult or painful menstruation? a. dysmenorrhea b. dyspareunia c. dysuria d. menorrhagia

a. dysmenorrhea

all of the following statements are true of endovaginal imaging except: a. endovaginal imaging requires a full urinary bladder b. endovaginal imaging leads to reduced waiting time for the patient and quicker medical management c. endovaginal imaging offers improved resolution of the endometrium, uterus, and ovaries, especially in the obese patient d. endovaginal imaging is contraindicated for pediatric patients, and for those with an intact hymen

a. endovaginal imaging requires a full urinary bladder

which of the following most often leads to an elevation of CA-125? a. ovarian carcinoma b. Fitz-Hugh-Curtis syndrome c. ovarian torsion d. ovarian hyperstimulation syndrome

a. ovarian carcinoma

which of the following would most likely be associated with hirsutism? a. polycystic ovary syndrome b. Meigs syndrome c. adenomyosis d. adenomyomatosis

a. polycystic ovary syndrome

precocious puberty is best defined as:

a. pubertal development before the age of 8

which of the following is defined as excessive hair growth in women in areas where hair growth is normally negligible?

b. hirsutism

which of the following would be best defined as abnormally heavy menstrual flow? a. menometrorrhagia b. menorrhgia c. metrorrhagia d. hypomenorrhea

b. menorrhagia

which of the following is best described as an artifact that is produced by a strong reflector and results in a copy of the anatomy being placed deeper than the correct location?

b. mirror image

which of the following diagnostic tests is used to evaluate emitted radiation from the patient to assess the function of organs? a. magnetic resonance imaging b. nuclear medicine c. radiography d. computed tomography

b. nuclear medicine

malignant ovarian tumors may leak mucinous material, and this condition is known as:

b. pseudomyxoma peritonei

which of the following definitions best describes the term adnexa? a. the area posterior to the uterus, between the uterus and rectum b. the area located posterior to the broad ligaments and adjacent to the uterus c. the area anterior to the uterus, between the uterus and urinary bladder d. the area lateral to the iliac crest and posterior to the pubic symphysis

b. the area located posterior to the broad ligaments and adjacent to the uterus

all of the following are associated with acute pelvic pain except: a. pelvic inflammatory disease b. ruptured ovarian hemorrhagic cyst c. perforated intrauterine contraceptive device d. Asherman syndrome

d. Asherman syndrome

what term relates to the number of pregnancies a patient has had? a. para b. menarche c. menorrhagia d. gravida

d. gravida

which of the following would be best defined as regularly timed menses but light flow? a. menometrorrhagia b. menorrhgia c. metrorrhagia d. hypomenorrhea

d. hypomenorrhea

which of the following is best defined as intermenstrual bleeding? a. dysmenorrhea b. menorrhagia c. menometrorrhagia d. metrorrhagia

d. metrorrhagia

leukocytosis would most likely be associated with: a. multiple degenerating fibroids b. ovarian teratoma c. adenomyosis d. pelvic inflammatory disease

d. pelvic inflammatory disease

the best way to communicate with a patient who speaks a language other than your own is to?

d. use a trained medical interpreter

Painful or difficult urination

dysuria

Anterior cul-de-sac

peritoneal outpouching located between the bladder and the uterus; also referred to as the vesicouterine pouch

occurs after menstruation and lasts until ovulation

proliferative phase

fluid accumulation within the uterus and vagina

Hydrometrocolpos

What ovarian timor will most likely have a moth-eaten appearance on sonography? A. Cystic teratoma B. Serous cystadenocarcinoma C. Krukenberg tumor D. Sertoli- Leydig cell tumor

C. Krukenberg tumor

Which of the following is the correct formula for calculating ovarian volume? A. L x W x H x 0.6243 B. L x W x H x 0.3899 C. L x W x H x 0.5233 D. Ovarian volume cannot be calculated

C. L x W x H x 0.5233

Pelvic muscle located posteriorly within the pelvis that helps support the sacrum

Coccygeus

During a pelvic sonogram you visualize a small cyst located adjacent to the ovary. What is the most likely etiology of this cyst? A. Dermoid Cyst B. Ovarian cystadenoma C. Endometrioma D. Paraovarian Cyst

D. Paraovarian Cyst

What structure within the female pelvis lies posterior to the urinary bladder and anterior to the rectum? a. Broad ligament b. Rectus abdominus muscle c. Space of Retzius d. Uterus

D. Uterus

A patient with an ovarian mass presents with an elevated serum AFP. Which of the following would be most likely? A. Ovarian fibroma B. Ovarian thecoma C. Cystic teratoma D. Yolk-sac tumor

D. Yolk-sac tumor

Intramural for pedunculated fibroids that tourists what are the clinical findings?

Acute, localized pelvic pain

Hormone of the anterior pituitary gland that causes the development of multiple follicles on the ovaries

Follicle-stimulating hormone

A benign cyst located within the vagina

Gartner duct cyst

What are the sonography findings of a fibroma?

Hypoechoic solid Mass with posterior enhancement, no posterior enhancement, if large mimic a pedunculated leiomyoma


Related study sets

Principles of Marketing Chpt. 1 Quiz

View Set

MIS112 Module 1-4, MIS112 Module 5-8

View Set

Managerial Leadership | Chapter 7 Test Bank

View Set

Fundamentals of Nursing Care Chapter 35 Medication Administration-Questions

View Set

Psychology 101 Week 9 - Consciousness

View Set

Personal Finance - Insurance Post Test

View Set

THINK Social Psychology 1st edition by Kimberly Duff. Chapter 6.

View Set

Psychology Themes and Variations Chapter 6

View Set